From 386e450b86fc334aa2264e5a68cf0d693dcbc0f4 Mon Sep 17 00:00:00 2001 From: "W. Trevor King" Date: Wed, 26 Aug 2009 17:10:32 -0400 Subject: [PATCH] Added rec10 solutions. --- .../Young_and_Freedman_12/problem29.01.tex | 36 ++++++ .../Young_and_Freedman_12/problem29.10.tex | 78 ++++++++++++ .../Young_and_Freedman_12/problem29.21.tex | 111 ++++++++++++++++++ .../Young_and_Freedman_12/problem29.25.tex | 107 +++++++++++++++++ .../Young_and_Freedman_12/problem29.28.tex | 56 +++++++++ .../Young_and_Freedman_12/problem29.53.tex | 82 +++++++++++++ 6 files changed, 470 insertions(+) create mode 100644 latex/problems/Young_and_Freedman_12/problem29.01.tex create mode 100644 latex/problems/Young_and_Freedman_12/problem29.10.tex create mode 100644 latex/problems/Young_and_Freedman_12/problem29.21.tex create mode 100644 latex/problems/Young_and_Freedman_12/problem29.25.tex create mode 100644 latex/problems/Young_and_Freedman_12/problem29.28.tex create mode 100644 latex/problems/Young_and_Freedman_12/problem29.53.tex diff --git a/latex/problems/Young_and_Freedman_12/problem29.01.tex b/latex/problems/Young_and_Freedman_12/problem29.01.tex new file mode 100644 index 0000000..709d83c --- /dev/null +++ b/latex/problems/Young_and_Freedman_12/problem29.01.tex @@ -0,0 +1,36 @@ +\begin{problem*}{29.1} % induction +A flat, rectangular coil consisting of $50$ turns measures +$25.0\U{cm}$ by $30.0\U{cm}$. It is in a uniform, $1.20\U{T}$, +magnetic field, with the plane of the coil parallel to the field. In +$0.222\U{s}$, it is rotated so that the plane of the coil is +perpendicular to the field. \Part{a} What is the change in the +magnetic flux through the coil due to this rotation? \Part{b} Find +the magnitude of the average emf induced in the coil during this +rotation. +\end{problem*} + +\begin{solution} +\Part{a} +Neither the area not the magnetic field changes, only the angle +between them, so +\begin{equation} + \Delta \Phi = AB[\cos(\theta_f)-\cos(\theta_i)] + = AB[\cos(0\dg)-\cos(90\dg)] + = AB = NLWB + = 50\cdot0.250\U{m}\cdot0.300\U{m}\cdot1.20\U{T} + = \ans{4.50\U{Wb}} \;. +\end{equation} + +\Part{b} +From Faraday's law of inductance (Eq.~29.3, p.~996) +\begin{equation} + \EMF = -\pderiv{t}{\Phi_B} \;, +\end{equation} +so the magnitude of the average \EMF\ is +\begin{equation} + |\EMF| = \frac{|\Delta \Phi|}{\Delta t} + = \frac{2.50\U{Wb}}{0.222\U{s}} + = \ans{20.3\U{V}} \;. +\end{equation} + +\end{solution} diff --git a/latex/problems/Young_and_Freedman_12/problem29.10.tex b/latex/problems/Young_and_Freedman_12/problem29.10.tex new file mode 100644 index 0000000..a671c16 --- /dev/null +++ b/latex/problems/Young_and_Freedman_12/problem29.10.tex @@ -0,0 +1,78 @@ +\begin{problem*}{29.10} % induction +A rectangle measuring $30.0\U{cm}$ by $40.0\U{cm}$ is located inside a +region of a spatially uniform magnetic field of $1.25\U{T}$, with the +field perpendicular to the plane of the coil (Fig.~29.29). The coil +is pulled out at a steady rate of $2.00\U{cm/s}$ traveling +perpendicular to the field lines. The region of field ends abruptly +as shown. Find the emf induced in this coil when it is \Part{a} all +inside the field; \Part{b} partly inside the field; \Part{c} all +outside the field. +\begin{center} +\begin{asy} +import Mechanics; +import ElectroMag; + +real u = 0.5cm; +real height = 4u; +real width = 3u; + +Vector Bs[]; +real dx = 1.1u; +real dy = dx; +int n = (int)(2*width / dx); +int m = (int)(2*height / dy); +real xstart = -width + (dx+fmod(2*width,dx))/2.0; +real ystart = -height + (dy+fmod(2*height,dy))/2.0; +for (int i=0; i0$. Thus the current direction is \ans{counter-clockwise} +($b\rightarrow a$). +% solutions say "clockwise" + +\Part{c} +The current in the circuit will be +\begin{align} + V &= IR \\ + I &= \frac{V}{R} + = \frac{3.00\U{V}}{1.50\U{\Ohm}} = 2.00\U{A} +\end{align} +You must then pull the bar with enough force to balance the magnetic +force +\begin{equation} + F = IHB = \frac{V}{R}HB = \frac{HBv}{R}HB = \frac{H^2B^2v}{R} + = \ans{0.800\U{N}} +\end{equation} + +\Part{d} +The mechanical power you put into the bar is given by +\begin{equation} + P_M = Fv = \frac{H^2B^2v^2}{R} \;. +\end{equation} +The electrical power dissipated by the resistance is +\begin{equation} + P_R = IV = \frac{V^2}{R} = \frac{H^2B^2v^2}{R} = P_M \;. +\end{equation} +This is good, because we expect energy to be conserved, and the +circuit contains no capacitors or other means of storing energy. +Therefore, what goes in must come out. +\end{solution} diff --git a/latex/problems/Young_and_Freedman_12/problem29.28.tex b/latex/problems/Young_and_Freedman_12/problem29.28.tex new file mode 100644 index 0000000..57c77b0 --- /dev/null +++ b/latex/problems/Young_and_Freedman_12/problem29.28.tex @@ -0,0 +1,56 @@ +\begin{problem*}{29.28} % ? +A long, thin solenoid has $900$ turns per meter and radius +$2.50\U{cm}$. The current in the solenoid is increasing at a uniform +rate of $60.0\U{A/s}$. What is the magnitude of the induced electric +field at a point near the center of the solenoid and \Part{a} +$0.500\U{cm}$ from the axis of the solenoid; \Part{b} $1.00\U{cm}$ +from the axis of the solenoid? +\end{problem*} + +\begin{solution} +For any position inside the solenoid ($r